A generic doubt in CR.. Experts please help..

This topic has expert replies
Senior | Next Rank: 100 Posts
Posts: 39
Joined: Tue Jul 20, 2010 5:17 am
An ingredient in marijuana known as THC has been found to inactivate herpesviruses in experiments. In previous experiments researchers found that inactivated herpesviruses can convert healthy cells into cancer cells. It can be concluded that the use of marijuana can cause cancer.

Which one of the following, if true, most seriously
weakens the argument?

(A) Several teams of scientists performed the various
experiments and all of the teams had similar
results.
(B) The carcinogenic effect of THC could be
neutralized by the other ingredients found in
marijuana.
(C) When THC kills herpesviruses it weakens the
immune system, and it might thus diminish the
body's ability to fight other viruses, including
viruses linked to cancers.
(D) If chemists modify the structure of THC, THC
can be safely incorporated into medications to
prevent herpes.
(E) To lessen the undesirable side effects of
chemotherapy, the use of marijuana has been
recommended for cancer patients who are free of
the herpesvirus.
More than the answer I am trying to get my concept right !!!!!

First the questions states "Which one of the following, if true, most seriously
weakens the argument? "
So here as the questions states all are true none of the options are far fetched.
Had the question been Which one of the following, most seriously weakens the argument?, wouldn't option B be far fetched as nothing is being said about any counteractive ingredient in marijuana. Correct me if I am wrong

Second I fell for option C, the reasoning behind this is TMH only weakens the immune system but something else that is carciogenic is making use of this. SO TMH in effect is not carciogenic

User avatar
Legendary Member
Posts: 1893
Joined: Sun May 30, 2010 11:48 pm
Thanked: 215 times
Followed by:7 members

by kvcpk » Sun Aug 08, 2010 6:03 am
I do not know what is the source of this. But its a very good question.

For Any passage, finding the conclusion and structure is important.
Here, the conclusion is:
the use of marijuana can cause cancer (M-> CC)

How is the author coming to this connclusion? Based on 2 premises:
1. An Ingredient in Marijuana inactivates herpesviruses (M->IH)
2. Inactivated herpes viruses can convert healthy cells to cancer cells. (IH -> CC)

Now look at the structure:
M-> IH
IH -> CC
Conclusion is saying M-> CC
This logically correct. We are asked to weaken this. Which means We need to prove M Does not Imply CC.
Premises are true. Answer choice is true. We shouldnt question them.
So, Only option left is to say that M might Imply IH. But M also has capacaity to negate CC.
This is what Option B does.

Let us look at option C:
The first statement of C itself doesnt look good to me. I eliminated it right after that.
When THC kills herpesviruses...
The passage doesnt say that THC kills them. It just says that it inactivates them.

One more thing is Conclusion is about Marijuana (not about THC in Marijuana). So Answer choice will most probably speak about Marijuana.

Keeping these reasons apart,
THC Weakens Immune system. This implies that the body is much prone to cancer.
This is supporting the Conclusion to an extent.
Conclusion says "CAN" cause cancer. So Option C "CAN" cause Cancer because immune system is weakened.

Hope this helps!!

Newbie | Next Rank: 10 Posts
Posts: 3
Joined: Fri Dec 24, 2010 10:42 pm

by aakash_saxena » Thu Jul 26, 2012 7:53 pm
Can you tell why E is incorrect ?

Doctor are using it for cure against cancer, then there must be something in it which neutralizes the bad effect of it. no ?

User avatar
Legendary Member
Posts: 502
Joined: Tue Jun 03, 2008 11:36 pm
Thanked: 99 times
Followed by:21 members

by vk_vinayak » Thu Jul 26, 2012 9:15 pm
aakash_saxena wrote:Can you tell why E is incorrect ?

Doctor are using it for cure against cancer, then there must be something in it which neutralizes the bad effect of it. no ?
Argument: M inactivates H viruses, and inactivated H viruses cause cancer. So, use of M causes cancer.

So, the assumption is that the only M can cause cancer is by inactivating H viruses. So the argument, in effect, is saying that if there are no H viruses in a person, then Marijuana can't (or may not) cause cancer.

That's what E is saying. M is recommended for people with no H viruses for medical reasons.
(E) To lessen the undesirable side effects of chemotherapy, the use of marijuana has been recommended for cancer patients who are free of the herpesvirus.
Thus, E is strengthening.
- VK

I will (Learn. Recognize. Apply)

User avatar
GMAT Instructor
Posts: 578
Joined: Tue Aug 25, 2009 6:00 pm
Thanked: 136 times
Followed by:62 members

by KapTeacherEli » Fri Jul 27, 2012 11:52 am
manishankar wrote:
An ingredient in marijuana known as THC has been found to inactivate herpesviruses in experiments. In previous experiments researchers found that inactivated herpesviruses can convert healthy cells into cancer cells. It can be concluded that the use of marijuana can cause cancer.

Which one of the following, if true, most seriously
weakens the argument?

(A) Several teams of scientists performed the various
experiments and all of the teams had similar
results.
(B) The carcinogenic effect of THC could be
neutralized by the other ingredients found in
marijuana.
(C) When THC kills herpesviruses it weakens the
immune system, and it might thus diminish the
body's ability to fight other viruses, including
viruses linked to cancers.
(D) If chemists modify the structure of THC, THC
can be safely incorporated into medications to
prevent herpes.
(E) To lessen the undesirable side effects of
chemotherapy, the use of marijuana has been
recommended for cancer patients who are free of
the herpesvirus.
More than the answer I am trying to get my concept right !!!!!

First the questions states "Which one of the following, if true, most seriously
weakens the argument? "
So here as the questions states all are true none of the options are far fetched.
Had the question been Which one of the following, most seriously weakens the argument?, wouldn't option B be far fetched as nothing is being said about any counteractive ingredient in marijuana. Correct me if I am wrong

Second I fell for option C, the reasoning behind this is TMH only weakens the immune system but something else that is carciogenic is making use of this. SO TMH in effect is not carciogenic
Hi manishankar,

The correct explanation has already been handled, but I wanted to address your specific points.

First, strengthen/weaken question will ALWAYS say "...if true..." or something similar as part of the Question Stem. You never need to worry about whether a statement like (B) is likely or probably. You are assuming it's true, and you only need to answer as if (B) were true, however weird (B) might seem!

Second, when a question asks for a cause, don't assume they want the proximate cause. It doesn't matter how roundabout the method of carcinogenesis is. If smoking marijuana ultimately leads some people to get cancer they would not otherwise have gotten, then marijuana is the (or a) cause of that cancer.
Eli Meyer
Kaplan GMAT Teacher
Cambridge, MA
www.kaptest.com/gmat

ImageImageImage